Howdy, Stranger!

It looks like you're new here. If you want to get involved, click one of these buttons!

Logical Reasoning: assumption and weaken questions

agathamelanieagathamelanie Free Trial Member
Hi! I'm having some major troubles with assumption and "which will weaken the argument" questions...any tips would REALLY be appreciated at this point!

For assumption questions, I know that if the answer is false and the argument falls apart, then that answer is correct but I can't really seem to have enough time to do that for every response.

For weaken the argument questions, I'm not sure if I'm mistakenly focusing on the premises and should be focusing on just the arguments instead. I know I should be looking to destroy the relationship between the premises and the arguments, but I'm finding it difficult.

Again, many thanks in advance for any tips/advice!

Comments

  • blah170blahblah170blah Alum Inactive ⭐
    3545 karma
    Hey @agathamelanie, can you walk us through a weaken question, preferably one you got wrong? I'm curious as to what your reasoning process consists of.
  • harrismeganharrismegan Member
    2074 karma
    My advice would to honestly be go pick up a copy of The Trainer and read through their flaw section and how to solve. I found that, once I read that section and did their exercises, everything kind of just... opened up.
    For weakening questions I found it easier to... when you're stuck... really write out what the conclusion is, what the support is, and then go through each answer choice. Ask yourself: "Does this answer make the conclusion less likely to be true?"
  • agathamelanieagathamelanie Free Trial Member
    edited June 2015 13 karma
    Hi @blah170blah thanks so much for your offer!!

    Heres an example, Preptest 64 Section 1 question 13:

    [removed - please do not post sections from the LSAT on the forums!]

    I answered B but D is correct. I guess I focused on the study too much? I though that because many were previously injured, they would have more difficulty to perform jogging and could somehow produce more injuries? I realize my reasoning is quite flawed but I still don't see why D is correct.
  • blah170blahblah170blah Alum Inactive ⭐
    3545 karma
    Oooo I remember this question. So let's start with identifying the conclusion: "stretching doesn't help prevent injuries." Okay cool -- why does the author believe this? Well there's 2 groups that had the same number of injuries, even though some stretched and some didn't. The assumption is that these 2 groups are similar, right? Otherwise, we have every reason to believe that there is another contributing factor to why the group that stretched still got injuries, while the control group is more normal.

    (D) gets at this contributing factor. Maybe it's the case that the experimental group that stretched are stretching because they have been injured in the past, and maybe it's the case that the people who are injured in the past or more likely to get injured again. In "lawgic speak," I would classify (D) as a case of reverse causation, which is one of the ways to weaken a causal argument (others include "same cause --> no effect // no cause --> same effect"). In this case, it's that the injuries causes stretching to not be as effective not that stretching causes the prevention of injuries.

    I think (B) tries to do what (D) does, so I think you had the right instinct. However, it plays off an assumption -- that the group talked about in (B) is the same group talked about in the stimulus. We can't make this assumption, however, which would therefore make (B) irrelevant to the argument at hand. (D), however, applies to the group in the stimulus because we are talking about "habitual" stretchers.

    That being said, I got this question wrong my first time through because I think the assumption is kind of a hard to see and the argument is easy to believe. However, we know there is an assumption being made so it's imperative we find it.
  • agathamelanieagathamelanie Free Trial Member
    13 karma
    @blah170blah thank you for going through that response - it was actually quite helpful! its also quite a relief to know that someone else got it wrong too!
  • visualcreedvisualcreed Member Inactive ⭐
    326 karma
    Just to note, this question appears a lot. What I mean is this is basically A doesnt cause B because Group 1 does A and Group 2 doesn't and it comes out the results for both are B. The answers for these questions I found are always (I haven't done all the PTs but in my experience always) something that basically says a factor is causing Group 1 to do A and that factor could be the reason for the same results.

    This makes sense because for these we're always looking to attack the support relationship to the conclusion. The difference between B and D in this case is that B doesn't look at the support just the conclusion. I find that most (if not always) the answer will specifically mention both the support and conclusion. Sorry if none of this makes sense.
  • blah170blahblah170blah Alum Inactive ⭐
    3545 karma
    @visualcreed is spot on. There are a ton of weaken questions that are like resolve questions where the argument makes an invalid assumption using two groups as support. When this happens, I get super excited normally because it usually goes 1 of 2 ways. Either the groups are treated the same even though there may be some really important difference that affects the truth of the conclusion (like the one above) or the groups are treated differently when they make actually be similar, which again, affects the truth of the conclusion.

    Typically, the wrong answer choices will fall under these camps:
    1) will be factually incorrect
    2) will strengthen the argument rather than weaken
    3) will introduce a subject shift always introducing the word "other" (x affects something "other" than y)
    4) misses a crucial part of the argument that then makes the AC out of scope/ irrelevant (exactly what happened in B by not introducing the phrase "habitually stretch")
  • nicole.hopkinsnicole.hopkins Inactive Sage Inactive ⭐
    7965 karma
    (I hate to be a wet blanket with all of this awesome learning going on, but @agathamelanie — posting LSAC content is against 7sage TOS and LSAC licensing :'-( Please be careful of posting actual LSAT content [when in doubt, leave it out—reference to PT/Section/Q is enough for us to look up the material to answer your Q's! :D] ... we don't want to start off our careers as attorneys with violation of copyright law)
  • nicole.hopkinsnicole.hopkins Inactive Sage Inactive ⭐
    edited June 2015 7965 karma
    @agathamelanie said:
    many of those
    Also don't forget that "many" means "some" which is automatically kind of a weaksauce choice ... So, many ... 3 or 4 out of 1,000 ?
Sign In or Register to comment.